A box plot was made to represent the number of matches won by 14 participants in a tennis tournament. The box plot had the box shifted to the right so that the left tail was much longer than the right tail. Based on the box plot, which conclusion is correct? (A)The mean and median of matches won are equal.
(B)The mean of matches won is more than the median of matches won.
(C)Most of the participants won many matches, but there were some participants who won very few matches compared to the others.
(D)Most of the participants won very few matches, but there were some participants who won many matches compared to the others.

Answers

Answer 1
Answer:

Answer:

(C)- Most of the participants won many matches, but there were some participants who won very few matches compared to the others.

Step-by-step explanation:

The Box plot isshifted to the right this means it is right-skewed data thus left tail is much longer than right tail. Hence, Option (C) is correct.

Option (A) means than neither right tail is longer nor left, the box will remain in middle only. So it is incorrect.

Option (B) is incorrect because Box Plot doesn't give us idea about mean.

Option (D) is also not correct because if this happened then box will shifted to the left

Answer 2
Answer: The box plot had the box shifted to the right so that the left tail was much longer than the right tail. Based on the box plot, the conclusion that is correct is Most of the participants won very few matches, but there were some participants who won many matches compared to the others. The answer is letter D

Related Questions

"I am an odd number. I am more than 40. I am less than 42. I am ________."
X=?
Which of the following occurs within the solution process for ∛(5x-2)-∛4x=0A.)squaring both sides onceB.)squaring both sides twiceC.)cubing both sides onceD.)cubing both sides twice
Jerry ate 1,070 calories total at breakfast. He ate a cup of yogurt that had 150 calories in it and some granola. Each cup of granola has 230 calories. how many cups
What is 699,900 rounded to the nearest thousand

What is the ratio of 33/36

Answers

all you have to do is find a number that both of them is divisible by and divide it by them. for example, when you divide it by 3, you will get 11/12
11/12 Divide both by 3.

Tom gets $12 off a box of chocolates that had an original $48. what percentage is the discount?

Answers

The discount price is the amount off by the discount divided by the original price.

12 / 48 = .25

The discount is 25% off.
Hope that helped =)
$12 
original price $48.

12 ÷ 48 = .25

Turn to percent:-

.25 × 100 = 25
25%

25% was the percentage discount. 

Consider the polynomial, 5x 9x 2x11 6.

Answers

It would be the 11th degree because that is the highest power in the expression. usually an expression will be arranged from highest power to lowest power going left to right. that is not true for this case but it is good information to know. 

Pv=kt solve for v
can anyone help

Answers

Hi Walker

pv=kt

Divide both sides by p

pv/p=kt/p

v= kt/p

I hope that's help:0

A prism whose sides and faces are all congruent squares is a

Answers

That's a description of a cube.
A prism whose sides and faces are all congruent squares is CUBE.

What is the value of x in the equation 3x – 1/9 = 18, when y = 27?

Answers

Answer:

x = 7

Step-by-step explanation:

3x - 1/9y = 18

Put y as 27 and evaluate.

3x - 27/9 = 18

3x - 3 = 18

Add 3 on both sides.

3x = 21

Divide 3 on both sides.

x = 7

Answer:

Here you go mate!

Step-by-step explanation: